13
$\begingroup$

I have always found union closed conjecture very interesting. There is an arxiv article claiming the solve the problem. My background is not appropriate to read the proof. Is this proof verified or wrong? Any comment would be appreciated.

The article was submitted in 2015 and it is not too long(about ten pages). Thus, I expect that it should be verified until now.

$\endgroup$
6
  • 6
    $\begingroup$ In this question (closed) there was a rather painful discussion on a different paper by Vladimir Blinovsky, he himself also participated as Mikhail mathoverflow.net/questions/176538/… $\endgroup$ Apr 2, 2017 at 8:15
  • 1
    $\begingroup$ Identical question one year ago (but with no answers): math.stackexchange.com/questions/1765189 $\endgroup$
    – HeinrichD
    Apr 2, 2017 at 9:06
  • 3
    $\begingroup$ My first reaction to this question, upon looking at the paper: "Well, it's just a ten page paper. Why not just check it?". And then I scrolled down to the second page and horrified, closed the paper. $\endgroup$
    – tomasz
    Apr 2, 2017 at 22:04
  • 7
    $\begingroup$ Vladimir Blinovsky also recently posted a 3 page proof of the Riemann Hypothesis on the arXiv arxiv.org/abs/1703.03827 $\endgroup$
    – Tony Huynh
    Apr 3, 2017 at 23:54
  • 4
    $\begingroup$ I’m voting to close this question because its asking for the correctness of a preprint claiming to prove an open conjecture. Further, this question was used to try to justify asking about a RH claimed proof, which is clearly off-topic. $\endgroup$
    – David Roberts
    Feb 4, 2022 at 9:17

1 Answer 1

23
$\begingroup$

There is a recent thesis by André da Cruz Carvalho on this conjecture. It discusses the paper by Vladimir Blinovsky and shows why his proof does not work (page 16). NB: I have not read it carefully, and this refers to the first version of Blinovsky's paper.

$\endgroup$
3
  • 1
    $\begingroup$ Has anyone checked if the criticism still applies to the current and 7th version of the paper? $\endgroup$
    – HeinrichD
    Apr 2, 2017 at 19:16
  • 1
    $\begingroup$ I checked - not applies V.Blinovsky :) $\endgroup$
    – Mikhail
    Apr 13, 2017 at 17:13
  • $\begingroup$ There's now an 8th version of the paper $\endgroup$
    – David Roberts
    Feb 4, 2022 at 9:20

Not the answer you're looking for? Browse other questions tagged or ask your own question.